Инвариантность фермионного действия относительно преобразований Лоренца

Предположим, у меня есть лагранжиан

л "=" 1 2 г а б ψ ¯ а Г к к ψ б
и я хочу показать его инвариантность относительно бесконечно малых преобразований Лоренца
дельта ψ а "=" Λ м н Икс м н ψ а + 1 2 Λ м н Σ м н ψ а ,
дельта ψ ¯ а "=" Λ м н Икс м н ψ ¯ а 1 2 Λ м н ψ ¯ а Σ м н ,
где Λ м н являются компонентами бесконечно малого преобразования Лоренца и, следовательно, антисимметричны, и Σ м н являются образующими спинорного представления группы Лоренца. Я действую обычным путем и после некоторой алгебры получаю это
дельта л "=" 1 2 г а б Λ м н [ Икс м н ψ ¯ а Г к к ψ б + Икс м ψ ¯ а Г к к ( н ψ б ) + ψ ¯ а Г м н ψ б ] .
Это нужно записать как полную производную, но я не могу этого добиться. Например, если я попытаюсь
м ( Λ м н Икс н л ) ,
Я получаю первые два, но не третий срок. Кто-нибудь может сказать мне, как действовать?

Вы включили в свою алгебру преобразование ковариантной производной? мю Λ мю ν ν
Привет СКФТ. Если вы еще этого не сделали, пожалуйста, найдите минутку, чтобы прочитать определение того, когда использовать тег « домашняя работа и упражнения» , и политику Phys.SE для проблем, подобных домашней работе.

Ответы (1)

В комментарии @Peter Anderson было указано, что вы забыли преобразование производной, которое в бесконечно малой форме должно читаться

дельта н "=" г л м Λ м н л
которое следует из преобразования Лоренца
н г л м ( л 1 ) м н л
(метрика предназначена для того, чтобы индексы согласовывались с выбором OP), который расширяется до
г л м ( л 1 ) м н "=" дельта н л г л м Λ м н + . . .
где я делаю вывод из других ваших законов преобразования, что вы применяете активные преобразования Лоренца, т.е. в символической перспективе
Икс л Икс
с полем, преобразующимся как
ф ( Икс ) М ф ( л 1 Икс )
существование М представление группы Лоренца.

Если вы воспользуетесь этим, вы получите новый член в вариации вашего лагранжиана.

дельта л 1 2 г а б Λ м н ψ ¯ а Г н г л м л ψ б
и этот новый термин с последним термином вашей вариации сохраняется
1 2 г а б Λ м н [ ψ ¯ а Г м н ψ б + ψ ¯ а Г н м ψ б ]
и поэтому у вас есть сокращение между антисимметричным тензором Λ м н с симметризованным количеством Г м н + Г н м и, следовательно, эти два термина исчезают. Затем у вас остаются первые два, которые, как вы уже сказали, можно переписать в полной производной форме.

Большое спасибо, что указали на это. Я принимал как должное, что дельта коммутирует с м , но в данном случае это не так.
Рад, что это помогло :) и да, в двух словах в этом случае это не коммутирует, поскольку производная не является скаляром Лоренца.